OlimpíadasSoma Telescópica Tópico resolvido

Aqui devem ser postados problemas Olímpicos. Informe a olimpíada e o ano no título do tópico. Exemplo: (OBM - 2008).

Moderador: [ Moderadores TTB ]

Avatar do usuário
Autor do Tópico
Santino
sênior
Mensagens: 43
Registrado em: Sex 19 Nov, 2021 17:18
Última visita: 20-02-23
Fev 2022 23 10:59

Soma Telescópica

Mensagem não lida por Santino »

Encontre A, B e C que tornam verdadeira a identidade [tex3]\frac{2n + 1}{n(n+1) (n + 2)} = \frac{A}{n} + \frac{B}{n + 1} + \frac{C}{n + 2}[/tex3] e utilize esse resultado para calcular o valor da soma [tex3]\frac{3}{1 . 2 . 3} + \frac{5}{2.3.4} + \frac{7}{3.4.5} + \frac{9}{5.6.7} + ...+\frac{2009}{1004.1005.1006}[/tex3]
O valor aproximado será igual a :

a) 1
b) 1,25
c) 1,5
d) 1,75
Resposta

b
Okay, minha dúvida é a seguinte : como faz a aplicação da soma telescópica ?
Anexos
gif.gif
gif.gif (835 Bytes) Exibido 1169 vezes

Última edição: Santino (Qua 23 Fev, 2022 11:08). Total de 3 vezes.



Avatar do usuário
LostWalker
4 - Sabe Tudo
Mensagens: 680
Registrado em: Seg 04 Mar, 2019 16:34
Última visita: 10-04-24
Fev 2022 23 12:36

Re: Soma Telescópica

Mensagem não lida por LostWalker »

Encontrando os Valores A, B, C
Imagino que a sua confusão se de aos sinais de [tex3](+)[/tex3] , ocorre que, achando os valores de A, B, C, pelo menos um será menos, possibilitando a soma telescópica. Vamos iniciar por:

[tex3]\frac{2n + 1}{n(n+1)(n + 2)}=\frac An+\frac B{n+1}+\frac C{n+2}[/tex3]


Vou te adiantar que inicialmente não é algo tão bonito, precisamos colocar todos na mesma base:

[tex3]\frac{2n + 1}{n(n+1)(n + 2)}=\frac{A(n+1)(n+2)+B(n)(n+2)+C(n)(n+1)}{n(n+1)(n + 2)}[/tex3]

[tex3]2n + 1=A(n+1)(n+2)+B(n)(n+2)+C(n)(n+1)[/tex3]


Agora precisamos evidenciar os valores em [tex3]n^2[/tex3] , [tex3]n[/tex3] e [tex3]n^0[/tex3] :

[tex3]2n + 1=n^2(A+B+C)+n(3A+2B+C)+(2A)[/tex3]


Agora fica mais fácil, sabemos que na direita, não tem termo [tex3]n^2[/tex3] , logo, [tex3](A+B+C)=0[/tex3] . De outro modo, tem termo [tex3]n[/tex3] , então [tex3]2n=n(3A+2B+C)[/tex3] , por último, [tex3]1=2A[/tex3]

Montamos então o sistema:

[tex3]\begin{cases}0=A+B+C\\2=3A+2B+C\\1=2A\end{cases}[/tex3]

[tex3]\begin{cases}0=A+B+C\\2=3A+2B+C\\A=\frac12\end{cases}[/tex3]

[tex3]\begin{cases}0=A+B+C\\2=3A+2B+C\end{cases}[/tex3]

o de baixo menos o de cima*

[tex3]2=2\cdot{\color{PineGreen}A}+B[/tex3]

[tex3]2=2\cdot{\color{PineGreen}\frac12}+B[/tex3]

[tex3]B=1[/tex3]


Voltando na primeira:

[tex3]0=A+B+C[/tex3]

[tex3]0=2A+2B+2C[/tex3]

[tex3]0=1+2\cdot1+2C[/tex3]

[tex3]C=-\frac32[/tex3]


Com isso temos que:

[tex3]\frac{2n + 1}{n(n+1)(n + 2)}=\frac An+\frac B{n+1}+\frac C{n+2}[/tex3]

[tex3]\frac{2n + 1}{n(n+1)(n + 2)}=\frac{\frac12}{n}+\frac 1{n+1}-\frac {\frac 32}{(n+2)}[/tex3]


Nisso temos uma soma telescópica sendo (não das mais bonitas).




Soma Telescópica
Como essa soma telescópica não é nem um pouco bonita, eu vou dividir ela em partes, temos então:

[tex3]\sum_{n=1}^{1004}\frac{2n + 1}{n(n+1)(n + 2)}=~~\sum_{n=1}^{1004}\frac{\frac12}{n}~~+~~\sum_{n=1}^{1004}\frac 1{n+1}~~-~~\sum_{n=1}^{1004}\frac {\frac 32}{n+2}[/tex3]

[tex3]=~~\frac12\sum_{n=1}^{1004}\frac{1}{n}~~+~~\sum_{n=1}^{1004}\frac 1{n+1}~~-~~\frac 32\sum_{n=1}^{1004}\frac {1}{n+2}[/tex3]


Vamos agora realizar mudanças nos limites de soma:

[tex3]=~~\frac12\sum_{\color{PineGreen}n=0}^{\color{PineGreen}1003}\frac{1}{\color{PineGreen}n+1}~~+~~\sum_{n=1}^{1004}\frac 1{n+1}~~-~~\frac 32\sum_{\color{Magenta}n=2}^{\color{Magenta}1005}\frac {1}{\color{Magenta}n+1}[/tex3]


Agora irei tirar valores específicos dos somatórios:

[tex3]=~~\frac12\sum_{\color{Purple}n=2}^{1003}\frac{1}{n+1}~~+~~\sum_{\color{NavyBlue}n=2}^{\color{NavyBlue}1003}\frac 1{n+1}~~-~~\frac 32\sum_{n=2}^{\color{Brown}1003}\frac {1}{n+1}{\color{Purple}~~+~~\frac12\sum_{n=0}^1\frac1{n+1}}{\color{NavyBlue}~~+~~\sum_{n=1}^1\frac1{n+1}~~+~\sum_{n=1004}^{1004}\frac1{n+1}}{\color{Brown}~~-\frac32\sum_{n=1004}^{1005}\frac1{n+1}}[/tex3]


Colocando em evidência a primeira parte:

[tex3]=~~{\color{Red}\frac12\sum_{n=2}^{1003}\frac{1}{n+1}~~+~~\sum_{n=2}^{1003}\frac 1{n+1}~~-~~\frac 32\sum_{n=2}^{1003}\frac {1}{n+1}}~~+~~\frac12\sum_{n=0}^1\frac1{n+1}~~+~~\sum_{n=1}^1\frac1{n+1}~~+~\sum_{n=1004}^{1004}\frac1{n+1}~~-\frac32\sum_{n=1004}^{1005}\frac1{n+1}[/tex3]

[tex3]=~~{\color{Red}\(\frac12+1-\frac32\)\sum_{n=2}^{1003}\frac{1}{n+1}}~~+~~\frac12\sum_{n=0}^1\frac1{n+1}~~+~~\sum_{n=1}^1\frac1{n+1}~~+~\sum_{n=1004}^{1004}\frac1{n+1}~~-\frac32\sum_{n=1004}^{1005}\frac1{n+1}[/tex3]

[tex3]=~~{\color{Red}\cancel{\color{Black}(0)\sum_{n=2}^{1003}\frac{1}{n+1}}}~~+~~\frac12\sum_{n=0}^1\frac1{n+1}~~+~~\sum_{n=1}^1\frac1{n+1}~~+~\sum_{n=1004}^{1004}\frac1{n+1}~~-\frac32\sum_{n=1004}^{1005}\frac1{n+1}[/tex3]

[tex3]=~~\frac12\sum_{n=0}^1\frac1{n+1}~~+~~\sum_{n=1}^1\frac1{n+1}~~+~\sum_{n=1004}^{1004}\frac1{n+1}~~-\frac32\sum_{n=1004}^{1005}\frac1{n+1}[/tex3]


Da para juntar umas partes aí? Dá... precisa? Não. Vamos só substituir os valores então:

[tex3]=~~\frac12\sum_{n=0}^1\frac1{n+1}~~+~~\sum_{n=1}^1\frac1{n+1}~~+~{\color{Red}\cancel{\color{Black}\sum_{n=1004}^{1004}\frac1{n+1}}^0}~~-{\color{Red}\cancel{\color{Black}\frac32\sum_{n=1004}^{1005}\frac1{n+1}}^0}[/tex3]

[tex3]=~~{\color{PineGreen}\frac12\sum_{n=0}^1\frac1{n+1}}~~+~~{\color{Purple}\sum_{n=1}^1\frac1{n+1}}[/tex3]

[tex3]={\color{PineGreen}\frac12\(\frac11+\frac12\)}+{\color{Purple}\frac1{2}}[/tex3]

[tex3]=\frac34+\frac12[/tex3]

[tex3]=\frac54[/tex3]


[tex3]\color{MidNightBlue}\boxed{\sum_{n=1}^{1004}\frac{2n + 1}{n(n+1)(n + 2)}\approx1.25}[/tex3]

[tex3]\color{MidNightBlue}\mbox{Alternativa B}[/tex3]



nota: deve haver meios mais fáceis de manipular as somas, mas eu não tenho tanto conhecimento em somatórios, então fui por um caminho mais lento



"[...] Mas essa é a graça dos encontros e desencontros: a Coincidência e o Destino. Se pudesse resumir, diria: A causalidade é a Ironia do Universo."
-Melly

Avatar do usuário
Autor do Tópico
Santino
sênior
Mensagens: 43
Registrado em: Sex 19 Nov, 2021 17:18
Última visita: 20-02-23
Fev 2022 23 18:19

Re: Soma Telescópica

Mensagem não lida por Santino »

LostWalker escreveu:
Qua 23 Fev, 2022 12:36
Encontrando os Valores A, B, C
Imagino que a sua confusão se de aos sinais de [tex3](+)[/tex3] , ocorre que, achando os valores de A, B, C, pelo menos um será menos, possibilitando a soma telescópica. Vamos iniciar por:

[tex3]\frac{2n + 1}{n(n+1)(n + 2)}=\frac An+\frac B{n+1}+\frac C{n+2}[/tex3]


Vou te adiantar que inicialmente não é algo tão bonito, precisamos colocar todos na mesma base:

[tex3]\frac{2n + 1}{n(n+1)(n + 2)}=\frac{A(n+1)(n+2)+B(n)(n+2)+C(n)(n+1)}{n(n+1)(n + 2)}[/tex3]

[tex3]2n + 1=A(n+1)(n+2)+B(n)(n+2)+C(n)(n+1)[/tex3]


Agora precisamos evidenciar os valores em [tex3]n^2[/tex3] , [tex3]n[/tex3] e [tex3]n^0[/tex3] :

[tex3]2n + 1=n^2(A+B+C)+n(3A+2B+C)+(2A)[/tex3]


Agora fica mais fácil, sabemos que na direita, não tem termo [tex3]n^2[/tex3] , logo, [tex3](A+B+C)=0[/tex3] . De outro modo, tem termo [tex3]n[/tex3] , então [tex3]2n=n(3A+2B+C)[/tex3] , por último, [tex3]1=2A[/tex3]

Montamos então o sistema:

[tex3]\begin{cases}0=A+B+C\\2=3A+2B+C\\1=2A\end{cases}[/tex3]

[tex3]\begin{cases}0=A+B+C\\2=3A+2B+C\\A=\frac12\end{cases}[/tex3]

[tex3]\begin{cases}0=A+B+C\\2=3A+2B+C\end{cases}[/tex3]

o de baixo menos o de cima*

[tex3]2=2\cdot{\color{PineGreen}A}+B[/tex3]

[tex3]2=2\cdot{\color{PineGreen}\frac12}+B[/tex3]

[tex3]B=1[/tex3]


Voltando na primeira:

[tex3]0=A+B+C[/tex3]

[tex3]0=2A+2B+2C[/tex3]

[tex3]0=1+2\cdot1+2C[/tex3]

[tex3]C=-\frac32[/tex3]


Com isso temos que:

[tex3]\frac{2n + 1}{n(n+1)(n + 2)}=\frac An+\frac B{n+1}+\frac C{n+2}[/tex3]

[tex3]\frac{2n + 1}{n(n+1)(n + 2)}=\frac{\frac12}{n}+\frac 1{n+1}-\frac {\frac 32}{(n+2)}[/tex3]


Nisso temos uma soma telescópica sendo (não das mais bonitas).




Soma Telescópica
Como essa soma telescópica não é nem um pouco bonita, eu vou dividir ela em partes, temos então:

[tex3]\sum_{n=1}^{1004}\frac{2n + 1}{n(n+1)(n + 2)}=~~\sum_{n=1}^{1004}\frac{\frac12}{n}~~+~~\sum_{n=1}^{1004}\frac 1{n+1}~~-~~\sum_{n=1}^{1004}\frac {\frac 32}{n+2}[/tex3]

[tex3]=~~\frac12\sum_{n=1}^{1004}\frac{1}{n}~~+~~\sum_{n=1}^{1004}\frac 1{n+1}~~-~~\frac 32\sum_{n=1}^{1004}\frac {1}{n+2}[/tex3]


Vamos agora realizar mudanças nos limites de soma:

[tex3]=~~\frac12\sum_{\color{PineGreen}n=0}^{\color{PineGreen}1003}\frac{1}{\color{PineGreen}n+1}~~+~~\sum_{n=1}^{1004}\frac 1{n+1}~~-~~\frac 32\sum_{\color{Magenta}n=2}^{\color{Magenta}1005}\frac {1}{\color{Magenta}n+1}[/tex3]


Agora irei tirar valores específicos dos somatórios:

[tex3]=~~\frac12\sum_{\color{Purple}n=2}^{1003}\frac{1}{n+1}~~+~~\sum_{\color{NavyBlue}n=2}^{\color{NavyBlue}1003}\frac 1{n+1}~~-~~\frac 32\sum_{n=2}^{\color{Brown}1003}\frac {1}{n+1}{\color{Purple}~~+~~\frac12\sum_{n=0}^1\frac1{n+1}}{\color{NavyBlue}~~+~~\sum_{n=1}^1\frac1{n+1}~~+~\sum_{n=1004}^{1004}\frac1{n+1}}{\color{Brown}~~-\frac32\sum_{n=1004}^{1005}\frac1{n+1}}[/tex3]


Colocando em evidência a primeira parte:

[tex3]=~~{\color{Red}\frac12\sum_{n=2}^{1003}\frac{1}{n+1}~~+~~\sum_{n=2}^{1003}\frac 1{n+1}~~-~~\frac 32\sum_{n=2}^{1003}\frac {1}{n+1}}~~+~~\frac12\sum_{n=0}^1\frac1{n+1}~~+~~\sum_{n=1}^1\frac1{n+1}~~+~\sum_{n=1004}^{1004}\frac1{n+1}~~-\frac32\sum_{n=1004}^{1005}\frac1{n+1}[/tex3]

[tex3]=~~{\color{Red}\(\frac12+1-\frac32\)\sum_{n=2}^{1003}\frac{1}{n+1}}~~+~~\frac12\sum_{n=0}^1\frac1{n+1}~~+~~\sum_{n=1}^1\frac1{n+1}~~+~\sum_{n=1004}^{1004}\frac1{n+1}~~-\frac32\sum_{n=1004}^{1005}\frac1{n+1}[/tex3]

[tex3]=~~{\color{Red}\cancel{\color{Black}(0)\sum_{n=2}^{1003}\frac{1}{n+1}}}~~+~~\frac12\sum_{n=0}^1\frac1{n+1}~~+~~\sum_{n=1}^1\frac1{n+1}~~+~\sum_{n=1004}^{1004}\frac1{n+1}~~-\frac32\sum_{n=1004}^{1005}\frac1{n+1}[/tex3]

[tex3]=~~\frac12\sum_{n=0}^1\frac1{n+1}~~+~~\sum_{n=1}^1\frac1{n+1}~~+~\sum_{n=1004}^{1004}\frac1{n+1}~~-\frac32\sum_{n=1004}^{1005}\frac1{n+1}[/tex3]


Da para juntar umas partes aí? Dá... precisa? Não. Vamos só substituir os valores então:

[tex3]=~~\frac12\sum_{n=0}^1\frac1{n+1}~~+~~\sum_{n=1}^1\frac1{n+1}~~+~{\color{Red}\cancel{\color{Black}\sum_{n=1004}^{1004}\frac1{n+1}}^0}~~-{\color{Red}\cancel{\color{Black}\frac32\sum_{n=1004}^{1005}\frac1{n+1}}^0}[/tex3]

[tex3]=~~{\color{PineGreen}\frac12\sum_{n=0}^1\frac1{n+1}}~~+~~{\color{Purple}\sum_{n=1}^1\frac1{n+1}}[/tex3]

[tex3]={\color{PineGreen}\frac12\(\frac11+\frac12\)}+{\color{Purple}\frac1{2}}[/tex3]

[tex3]=\frac34+\frac12[/tex3]

[tex3]=\frac54[/tex3]


[tex3]\color{MidNightBlue}\boxed{\sum_{n=1}^{1004}\frac{2n + 1}{n(n+1)(n + 2)}\approx1.25}[/tex3]

[tex3]\color{MidNightBlue}\mbox{Alternativa B}[/tex3]



nota: deve haver meios mais fáceis de manipular as somas, mas eu não tenho tanto conhecimento em somatórios, então fui por um caminho mais lento
Vlw pela resolução, muito bemm feita. Agora, uma última dúvida. Até o início da soma telescópica entendi muito bem, agr quando começa a cancelar...aí que me pegou. Como esses cancelamentos surgem ?
?



Avatar do usuário
LostWalker
4 - Sabe Tudo
Mensagens: 680
Registrado em: Seg 04 Mar, 2019 16:34
Última visita: 10-04-24
Fev 2022 23 21:26

Re: Soma Telescópica

Mensagem não lida por LostWalker »

De maneira curta e grossa, o enunciado pediu um valor aproximado, aqueles números já eram tão pequenos que eram irrelevantes, esqueci de avisar issoSantino, sorry



"[...] Mas essa é a graça dos encontros e desencontros: a Coincidência e o Destino. Se pudesse resumir, diria: A causalidade é a Ironia do Universo."
-Melly

Movido de IME / ITA para Olimpíadas em Ter 15 Mar, 2022 12:17 por ALDRIN

Responder
  • Tópicos Semelhantes
    Respostas
    Exibições
    Última msg
  • Nova mensagem Cubo da soma
    por EinsteinGenio » » em Ensino Médio
    2 Respostas
    502 Exibições
    Última msg por petras
  • Nova mensagem PUC - Propriedade e Soma dos Termos de uma PA
    por Viytor » » em Ensino Médio
    1 Respostas
    4912 Exibições
    Última msg por petras
  • Nova mensagem Soma Infinita
    por NigrumCibum » » em IME / ITA
    1 Respostas
    794 Exibições
    Última msg por FelipeMartin
  • Nova mensagem Calculo arestas de 2 cubos tendo a soma e volume de um deles
    por Carlosft57 » » em Ensino Médio
    1 Respostas
    4432 Exibições
    Última msg por Carlosft57
  • Nova mensagem (Mackenzie-SP)- Soma vetorial
    por inguz » » em Física I
    1 Respostas
    4636 Exibições
    Última msg por careca

Voltar para “Olimpíadas”